LSAT and Law School Admissions Forum

Get expert LSAT preparation and law school admissions advice from PowerScore Test Preparation.

 Administrator
PowerScore Staff
  • PowerScore Staff
  • Posts: 8916
  • Joined: Feb 02, 2011
|
#91409
Complete Question Explanation

The correct answer choice is (A).

Answer choice (A): This is the correct answer choice.

Answer choice (B):

Answer choice (C):

Answer choice (D):

Answer choice (E):

This explanation is still in progress. Please post any questions below!
User avatar
 christinecwt
  • Posts: 74
  • Joined: May 09, 2022
|
#95545
Hi Team - may I know why Jazz trio is possible to be on May 1? Isn't that O must be ahead of J that J can never be on the first day? Thanks!
 Adam Tyson
PowerScore Staff
  • PowerScore Staff
  • Posts: 5153
  • Joined: Apr 14, 2011
|
#95573
The first rule means that whenever you have an O, you must have a J right after it. That does not mean that whenever you have a J there must be an O before it - that would be a Mistaken Reversal of that rule. Putting a J in the first position wouldn't violate that rule about what you have to do when you place an O.

The rule could be written like this:

O :arrow: OJ

The O is Sufficient, not the J. A J doesn't require you to do anything!
User avatar
 christinecwt
  • Posts: 74
  • Joined: May 09, 2022
|
#95788
Thanks a lot Adam :)

Get the most out of your LSAT Prep Plus subscription.

Analyze and track your performance with our Testing and Analytics Package.